www.vorhilfe.de
Vorhilfe

Kostenlose Kommunikationsplattform für gegenseitige Hilfestellungen.
Hallo Gast!einloggen | registrieren ]
Startseite · Forum · Wissen · Kurse · Mitglieder · Team · Impressum
Forenbaum
^ Forenbaum
Status Vorhilfe
  Status Geisteswiss.
    Status Erdkunde
    Status Geschichte
    Status Jura
    Status Musik/Kunst
    Status Pädagogik
    Status Philosophie
    Status Politik/Wirtschaft
    Status Psychologie
    Status Religion
    Status Sozialwissenschaften
  Status Informatik
    Status Schule
    Status Hochschule
    Status Info-Training
    Status Wettbewerbe
    Status Praxis
    Status Internes IR
  Status Ingenieurwiss.
    Status Bauingenieurwesen
    Status Elektrotechnik
    Status Maschinenbau
    Status Materialwissenschaft
    Status Regelungstechnik
    Status Signaltheorie
    Status Sonstiges
    Status Technik
  Status Mathe
    Status Schulmathe
    Status Hochschulmathe
    Status Mathe-Vorkurse
    Status Mathe-Software
  Status Naturwiss.
    Status Astronomie
    Status Biologie
    Status Chemie
    Status Geowissenschaften
    Status Medizin
    Status Physik
    Status Sport
  Status Sonstiges / Diverses
  Status Sprachen
    Status Deutsch
    Status Englisch
    Status Französisch
    Status Griechisch
    Status Latein
    Status Russisch
    Status Spanisch
    Status Vorkurse
    Status Sonstiges (Sprachen)
  Status Neuerdings
  Status Internes VH
    Status Café VH
    Status Verbesserungen
    Status Benutzerbetreuung
    Status Plenum
    Status Datenbank-Forum
    Status Test-Forum
    Status Fragwürdige Inhalte
    Status VH e.V.

Gezeigt werden alle Foren bis zur Tiefe 2

Navigation
 Startseite...
 Neuerdings beta neu
 Forum...
 vorwissen...
 vorkurse...
 Werkzeuge...
 Nachhilfevermittlung beta...
 Online-Spiele beta
 Suchen
 Verein...
 Impressum
Das Projekt
Server und Internetanbindung werden durch Spenden finanziert.
Organisiert wird das Projekt von unserem Koordinatorenteam.
Hunderte Mitglieder helfen ehrenamtlich in unseren moderierten Foren.
Anbieter der Seite ist der gemeinnützige Verein "Vorhilfe.de e.V.".
Partnerseiten
Dt. Schulen im Ausland: Mathe-Seiten:

Open Source FunktionenplotterFunkyPlot: Kostenloser und quelloffener Funktionenplotter für Linux und andere Betriebssysteme
Forum "Integrationstheorie" - Integration
Integration < Integrationstheorie < Maß/Integrat-Theorie < Analysis < Hochschule < Mathe < Vorhilfe
Ansicht: [ geschachtelt ] | ^ Forum "Integrationstheorie"  | ^^ Alle Foren  | ^ Forenbaum  | Materialien

Integration: Integration mit Lambda nach dt
Status: (Frage) beantwortet Status 
Datum: 20:11 So 01.01.2017
Autor: Elza57

Aufgabe
Die Dichte der Exponentialverteilung ist

[mm] $\int_{-\infty}^{x} {\phi(t) = \lambda e^{-\lambda t}} [/mm]

Damit errechnet sich die zugehörige kumulierte Wahrscheinlichkeitsverteilung zu

F(x) = [mm] \int_{-\infty}^{x} {\phi(t)~dt =} \int_{-\infty}^{x} {\lambda e^{-\lambda t} ~dt =} [/mm] 1 - [mm] e^{-\lambda x} [/mm]

Geben Sie die (kumulierte) Verteilungsfunktion der Exponentialverteilung für beliebigen Paramter [mm] ${\lambda > 0}$ [/mm] an.



Ich habe diese Frage bei onlinemathe auch gestellt:
http://www.onlinemathe.de/forum/Integration-einer-Gleichung-nach-t

Hallo Leute,

ich muss bis Dienstag eine Hausaufgabe abgeben, aber ich schaffe es leider nicht, auf das Endergebnis zu kommen. Mittlerweile befürchte ich, dass ich etwas fundamental falsch mache. Es wäre sehr freundlich, wenn Ihr mir zeigen könntet, wir ich zu diesem Ergebnis komme.

Mein Ansatz:


[mm] \int_{-\infty}^{x} \lambda e^{-\lambda t} [/mm] dt = [mm] [-e^{-\lambda t}]_{-\infty}^{x} [/mm] = [mm] -e^{-\lambda x} [/mm] - [mm] (-e^{-\lambda \star (-\infty})) [/mm] = [mm] -e^{-\lambda x} [/mm] - [mm] \frac{1}{(-e^{\lambda \star (\infty}))} [/mm] = [mm] -e^{-\lambda x} [/mm] - [mm] \frac{1}{-\infty} [/mm] = [mm] -e^{-\lambda x} [/mm] + 0 = [mm] -e^{-\lambda x} [/mm]

Ich erhalte also fast das selbe aus der Integration wie in der Lösung oben. Aber mir fehlt die 1. Was mache ich falsch? Ist mein Ansatz überhaupt richtig?

Liebe Grüße
Elza

        
Bezug
Integration: Antwort
Status: (Antwort) fertig Status 
Datum: 20:38 So 01.01.2017
Autor: Diophant

Hallo,

es kommt selten vor, dass eine Aufgabe samt Lösung eingestellt wird verbunden mit einer solchen Frage ohne jedweden eigenen Ansatz. Man kann so nicht verstehen, was dein Anliegen ist!

Beim Integrieren der Dichte musst du zwei Dinge beachten:

- [mm] \lambda [/mm] ist eine Konstante. Fasst man die Dichte als verkettete Funktion auf, so ist die innere Funktion linear. Wie das beim Integrieren gehandhabt wird, sollte aus der Schule bekannt sein.
-Verteilungsfunktionen haben einige einschlägig bekannte Eigenschaften, als da wären:

  - monoton steigend
  - für [mm] x->-\infty [/mm] streben sie gegen 0
  - für [mm] x->\infty [/mm] streben sie gegen 1

Achte darauf, dass dies gewährleistet ist.

Wenn du eine wirklich zielführende Hilfe haben möchtest, dann wirst du wohl deine Versuche, oder zumindest einen Versuch, hier zusammen mit konkreten Fragen präsentieren müssen.

Irgendwo in deinem Profil solltest du einstellen können, dass du an sog. 'Beta-Tests' teilnehmen möchtest. Das ist sehr kryptisch formuliert, wenn du es aktivierst steht dir zum Verfassen von Beiträgen ein LaTeX-Formeleditor zur Verfügung (und sonst bewirkt es m.W. auch nichts weiter). Damit kannst du recht einfach mathematische Notationen realisieren, um deine Rechnungen zu posten.

EDIT: ich habe in dieser Antwort deinen entscheidenden Fehler oben übersehen. Dazu mehr in meiner anderen Antwort.


Gruß und frohes neues Jahr, Diophant
    


Bezug
                
Bezug
Integration: Änderung
Status: (Mitteilung) Reaktion unnötig Status 
Datum: 21:30 So 01.01.2017
Autor: Elza57

Hallo Diophant!

Ich werde den Post entsprechend ändern. Hoffentlich kannst Du mir dann weiterhelfen.

Danke

Bezug
        
Bezug
Integration: Antwort
Status: (Antwort) fertig Status 
Datum: 22:33 So 01.01.2017
Autor: Diophant

Hallo,

jetzt wird dein Anliegen klarer, und ich muss mich dafür entschuldigen, dass ich vorhin etwas übersehen habe.

Die Dichte der Exponentialverteilung ist bei dir insofern falsch, als für x<0 natürlich [mm] \phi(t)=0 [/mm] gilt. Das bedeutet, dass dein Integral von 0 bis x geht und eben nicht von [mm] -\infty [/mm] bis x.

Berücksichtige das einmal in deiner Rechnung (die an einer Stelle falsch ist, aber das wird sich von selbst erledigen, wenn du meinem Hinweis folgst). Ich denke, das könnte das fehlende Stück im Puzzle sein.

Man sollte sich bei den unterschiedlichen Wahrscheinlichkeitsverteilungen stets im Klaren sein, welches Modell bzw. welche Annahmen dahinter stehen. Dann übersieht man so etwas wie diese falsche Integrationsgrenze nicht. Aber heute gelten sicherlich 'mildernde Umstände' in dieser Hinsicht.

Gruß, Diophant

Bezug
                
Bezug
Integration: Frage (beantwortet)
Status: (Frage) beantwortet Status 
Datum: 11:06 Di 03.01.2017
Autor: Elza57

Hallo Diophant,

danke für die Hilfe. Mit deinem Hinweis, dass [mm] \phi(t)=0 [/mm] ist, konnte ich auf das richtige Ergebnis kommen. Leider weiß ich aber nicht wieso das gilt. Kannst Du mir das erklären?

Grüße,
Elza

Bezug
                        
Bezug
Integration: Antwort
Status: (Antwort) fertig Status 
Datum: 12:15 Di 03.01.2017
Autor: fred97


> Hallo Diophant,
>  
> danke für die Hilfe. Mit deinem Hinweis, dass [mm]\phi(t)=0[/mm]
> ist, konnte ich auf das richtige Ergebnis kommen. Leider
> weiß ich aber nicht wieso das gilt. Kannst Du mir das
> erklären?
>  


So ist die Dichte  der Exponentialverteilung definiert!


> Grüße,
>  Elza


Bezug
        
Bezug
Integration: Mitteilung
Status: (Mitteilung) Reaktion unnötig Status 
Datum: 23:29 So 01.01.2017
Autor: donquijote

Hallo,
noch zwei Anmerkungen:

> Die Dichte der Exponentialverteilung ist
>  
> [mm]$\int_{-\infty}^{x} {\phi(t) = \lambda e^{-\lambda t}}[/mm]

Die Dichte ist [mm]\phi(t) = \lambda e^{-\lambda t}}[/mm] für [mm]t\ge 0[/mm] und [mm]\phi(t) = 0[/mm] für t<0 (ohne das Integral). Das Integral (mit unterer Grenze 0) macht dann daraus die Verteilungsfunktion.

>  
> Damit errechnet sich die zugehörige kumulierte
> Wahrscheinlichkeitsverteilung zu
>  
> F(x) = [mm]\int_{-\infty}^{x} {\phi(t)~dt =} \int_{-\infty}^{x} {\lambda e^{-\lambda t} ~dt =}[/mm]
> 1 - [mm]e^{-\lambda x}[/mm]
>  
> Geben Sie die (kumulierte) Verteilungsfunktion der
> Exponentialverteilung für beliebigen Paramter [mm]{\lambda > 0}[/mm]
> an.
>  
>
> Ich habe diese Frage bei onlinemathe auch gestellt:
>  
> http://www.onlinemathe.de/forum/Integration-einer-Gleichung-nach-t
>  
> Hallo Leute,
>  
> ich muss bis Dienstag eine Hausaufgabe abgeben, aber ich
> schaffe es leider nicht, auf das Endergebnis zu kommen.
> Mittlerweile befürchte ich, dass ich etwas fundamental
> falsch mache. Es wäre sehr freundlich, wenn Ihr mir zeigen
> könntet, wir ich zu diesem Ergebnis komme.
>  
> Mein Ansatz:
>  
>
> [mm]\int_{-\infty}^{x} \lambda e^{-\lambda t}[/mm] dt =
> [mm][-e^{-\lambda t}]_{-\infty}^{x}[/mm] = [mm]-e^{-\lambda x}[/mm] -
> [mm](-e^{-\lambda \star (-\infty}))[/mm] = [mm]-e^{-\lambda x}[/mm] -
> [mm]\frac{1}{(-e^{\lambda \star (\infty}))}[/mm] = [mm]-e^{-\lambda x}[/mm] -
> [mm]\frac{1}{-\infty}[/mm] = [mm]-e^{-\lambda x}[/mm] + 0 = [mm]-e^{-\lambda x}[/mm]

Abgesehen davon, dass die untere Grenze nicht korrekt ist, ist auch die Rechnung falsch. Für [mm]t\to-\infty[/mm] strebt die Stammfunktion gegen [mm]-\infty[/mm], so dass das uneigentliche Integral divergiert.

>  
> Ich erhalte also fast das selbe aus der Integration wie in
> der Lösung oben. Aber mir fehlt die 1. Was mache ich
> falsch? Ist mein Ansatz überhaupt richtig?
>  
> Liebe Grüße
>  Elza


Bezug
Ansicht: [ geschachtelt ] | ^ Forum "Integrationstheorie"  | ^^ Alle Foren  | ^ Forenbaum  | Materialien


^ Seitenanfang ^
www.vorhilfe.de